Question

Which of the following statements about Beta is true? 1. Betas are about equally likely to...

Which of the following statements about Beta is true?

1. Betas are about equally likely to be positive or negative

2. The average beta is about 1

3. The better diversified a portfolio is, the closer its beta is to 0

4. The expected return on a stock with a beta of 2.0 should be twice the expected return of a stock with a beta of 1.0.

a. 4 only

b. 3 and 4

c. 2 and 4

d. 2 only

e. 1 and 3

f. 1 and 4

Homework Answers

Answer #1

Beta

It is a tool or a standard to measure the systematic risk or volatility for portfolio or security.

Each of the above statements is explained as follows:

1. The beta of a stock is more likely to positive than negative. A negative beta is a highly unlikely situation. Therefore, this statement is False.

2. The market beta of a stock or security is around 1. This means the price of security tends to move with the market. Therefore the above statement is True.

3. It is not necessary that the better-diversified portfolio will have a beta tending towards zero. The above statement is False.

4. The returns of the stock are not affected with the change in beta since beta only measures the risk of a security or a portfolio. The above statement is False.

Option d) 2 only is the correct answer for the question asked above.

Please comment in case of any questions. Upvote if you like the answer.

Know the answer?
Your Answer:

Post as a guest

Your Name:

What's your source?

Earn Coins

Coins can be redeemed for fabulous gifts.

Not the answer you're looking for?
Ask your own homework help question
Similar Questions
2. Which of the following statements concerning beta is correct? a. A stock with a beta...
2. Which of the following statements concerning beta is correct? a. A stock with a beta of 0 is expected to provide a rate of return equal to the market portfolio b. A stock with a beta equal to 1 has no risk c. Stocks with negative betas have the least amount of risk FALSE d. A stock with a beta greater than 1 is expected to be more volatile than the market portfolio
Which of the following statements is CORRECT? Select one: a. The beta of a portfolio of...
Which of the following statements is CORRECT? Select one: a. The beta of a portfolio of stocks is always smaller than the betas of any of the individual stocks. b. The beta of a portfolio of stocks is always larger than the betas of any of the individual stocks. c. It is theoretically possible for a stock to have a beta of 1.0. If a stock did have a beta of 1.0, then, at least in theory, its required rate...
Which of the following statements about the beta coefficient is false? A A stock’s beta coefficient...
Which of the following statements about the beta coefficient is false? A A stock’s beta coefficient measures its volatility relative to the market portfolio. B A stock’s beta coefficient can be estimated by plotting the stock’s returns versus the market portfolio’s returns. C A stock’s reported beta coefficient is based on forecasted future volatility. D A stock with a beta coefficient greater than 1.0 is said to be riskier than the market portfolio. E Using the capital asset pricing model,...
Which of the following statements is most correct? The required rate of return of a diversified...
Which of the following statements is most correct? The required rate of return of a diversified portfolio with Beta of 1 is typically greater than the Market Risk Premium. A stock with a negrative beta must have a negative required rate of return. If a stock's beta doubles its required rate of return must double. If a stock has a beta equal to 1.0, its required rate of return will be unaffected by changes in the market risk premium. None...
Q1. Which of the following statements about the portfolio is true? a. The expected return of...
Q1. Which of the following statements about the portfolio is true? a. The expected return of a portfolio is NOT the weighted average of the expected returns of all individual stocks in the portfolio. b. The standard deviation of a portfolio is NOT the weighted average of the standard deviations of all individual stocks in the portfolio. c. Portfolio beta is NOT the weighted average of the beta values of all individual stocks in the portfolio Q2. Which of the...
1. Asset 1 has a beta of 1.2 and Asset 2 has a beta of 0.6....
1. Asset 1 has a beta of 1.2 and Asset 2 has a beta of 0.6. Which of the following statements is correct? A. Asset 1 is more volatile than Asset 2. B. Asset 1 has a higher expected return than Asset 2. C. In a regression with individual asset’s return as the dependent variable and the market’s return as the independent variable, the R-squared value is higher for Asset 1 than it is for Asset 2. D. All of...
Which of the following statements is true with respect to beta? Multiple Choice A - All...
Which of the following statements is true with respect to beta? Multiple Choice A - All of the above. B-- A stock with a beta > 1 is more volatile than the market portfolio. C -- The market portfolio has a beta of "0". D -- A stock with a beta < 1 will outperform the market portfolio when the market is up.
1. Beale Manufacturing Company has a beta of 1.2, and Foley Industries has a beta of...
1. Beale Manufacturing Company has a beta of 1.2, and Foley Industries has a beta of 0.9. The required return on an index fund that holds the entire stock market is 10.5%. The risk-free rate of interest is 7%. By how much does Beale's required return exceed Foley's required return? 2. Suppose you held a diversified portfolio consisting of a $7,500 investment in each of 20 different common stocks. The portfolio's beta is 1.83. Now suppose you decided to sell...
Stock A's beta is 1.5 and Stock B's beta is 0.5. Which of the following statements...
Stock A's beta is 1.5 and Stock B's beta is 0.5. Which of the following statements must be true, assuming the CAPM is correct. a. Stock A would be a more desirable addition to a portfolio then Stock B. b. Stock B would be a more desirable addition to a portfolio than A. c. When held in isolation, Stock A has more risk than Stock B. d. In equilibrium, the expected return on Stock B will be greater than that...
Stock A's beta is 1.5 and Stock B's beta is 0.5. Which of the following statements...
Stock A's beta is 1.5 and Stock B's beta is 0.5. Which of the following statements must be true, assuming the CAPM is correct. a. Stock A would be a more desirable addition to a portfolio then Stock B. b. In equilibrium, the expected return on Stock B will be greater than that on Stock A. c. When held in isolation, Stock A has more risk than Stock B. d. Stock B would be a more desirable addition to a...
ADVERTISEMENT
Need Online Homework Help?

Get Answers For Free
Most questions answered within 1 hours.

Ask a Question
ADVERTISEMENT